Translate

Thursday, July 30, 2015

Bài toán: Cho $n$ là số nguyên dương. Tìm số đa thức $P(x)$ có hệ số thực thuộc $\{ 0, 1, 2, 3\}$ thỏa mãn $P(2)=n$.
Lời giải: 
Giả sử: $$P(x)=a_0+a_1x+a_2x^2+a_3x^3+...., \; \text{với } a_i\in\{ 0, 1, 2, 3\}$$ Do $P(2)=n$ nên $$a_0+2a_1+4a_2+...+2^ia_i+...=n\;\;\;\;\;\;\;\;\;\;\;\;(*)$$ Ta xây dựng hàm sinh cho phương trình $(*)$: $$\begin{aligned} G(x)&=\prod_{i\ge 0}\left( 1+x^{2^k}+x^{2.{2^k}}+x^{3.2^k}\right)\\&=\prod_{i\ge 0}\dfrac{1-(x^{2^k})^4}{1-x^{2^k}}\\&=\prod_{i\ge 0}\dfrac{1-x^{2^{k+2}}}{1-x^{2^k}}\\&=\dfrac{1}{(1-x)(1-x^2)}\\&=\dfrac{1}{4(1-x)}+\dfrac{1}{4(1+x)}+\dfrac{1}{2(1-x)^2}=\sum_{i\ge 0}\left( \dfrac{1}{4}+\dfrac{1}{4}(-1)^i+\dfrac{1}{2}C^{i}_{i+1}\right) x^i\end{aligned}$$ Số đa thức thỏa mãn bài toán cũng chính là hệ số của $x^n$ trong khai triển trên: $$\left[\dfrac{2n+3+(-1)^n}{4}\right]=\left[\dfrac{n}{2}\right]+1$$

Sunday, July 26, 2015

Bài toán: (Olympic 30/04/2015 - Khối 11) Cho $a,b,c$ là các số nguyên tố. Đặt $x=a+b-c,y=a+c-b,z=b+c-a$. Giả sử rằng $x^2=y$ và hiệu $\sqrt{z}-\sqrt{y}$ là bình phương của một số nguyên tố. Tính giá trị biểu thức: $$T=(a+2)(b-10)(c+2)$$
Lời Giải: 
Gọi $p$ là số nguyên tố sao cho $$\begin{aligned}&\sqrt{z}-\sqrt{y}=p^2\\\Leftrightarrow &z+y-2\sqrt{yz}=p^4\\\Leftrightarrow &2c-2\sqrt{(c+a-b)(b+c-a)}=p^4\end{aligned}$$ Từ đây suy ra $2\;|\; p\Rightarrow p=2$. Ta có hệ: $$\begin{cases}c-\sqrt{(c+a-b)(b+c-a)}=8\\(a+b-c)^2=c+a-b\end{cases}$$ Ta biến đổi PT đầu: $$\begin{aligned}& (c+a-b)(a+b-c)=(c-8)^2\\\Leftrightarrow &c^2-(a-b)^2=(c-8)^2\\\Leftrightarrow & (c-8)^2+(a-b)^2=c^2\end{aligned}$$ Xem đây là phương trình Pythargores, chú ý rằng $\gcd{(c-8, c, a-b)}=1$ (vì $a,b,c$ nguyên tố) nên:

     + Trường hợp 1: $\begin{cases} c-8=2mn\\a-b=m^2-n^2\\c=m^2+n^2\end{cases}\; (m,n\in\mathbb{N})$ Do $c$ nguyên tố mà lúc này $2\;|\; c$ nên $c=2$. Như vậy: $$(a+b-2)^2=a-b+2$$ Vì $\gcd{(c-8, a-b, c)}=1$ nên $a,b$ khác tính chẵn lẻ. Tuy nhiên khi $a=2\vee b=2$ thì hệ đều không thỏa mãn.

     + Trường hợp 2: $\begin{cases}a-b=2mn\\c-8=m^2-n^2\\c=m^2+n^2\end{cases}(m,n\in\mathbb{N})$ Từ đây suy ra luôn $n=2\Rightarrow \begin{cases}c=m^2+4\\a-b=4m\end{cases}$ Thay vào hệ trên ta được: $$(a+b-m^2-4)^2=(m+2)^2$$
          $\bullet $ Khả năng 1: $a+b-m^2-4=m+2\Leftrightarrow a+b=m^2+m+6$. Kết hợp với $a-b=4m$ suy ra: $$\begin{cases}a=\dfrac{(m^2+5m+6}{2}\\b=\dfrac{m^2-3m+6}{2}\end{cases}$$ Đặt $m=2k+1,k\in\mathbb{Z}$ Suy ra $a=(k+2)(2k+3)$ Do $a$ nguyên tố nên $$2k+1=^+_- 1\vee k+2=^+_-1\Rightarrow k=-3\Rightarrow a=3, b=23, c=29\Rightarrow \boxed{T=2015}$$
     
         $\bullet $ Khả năng 2: $a+b-m^2-4=-m-2\Leftrightarrow a+b=m^2-m+2$. Suy ra: $$\begin{cases} a=\dfrac{m^2+2m+2}{2}\\b=\dfrac{m^2-5m+2}{2}\end{cases}$$ Đặt $m=2k+1, k\in\mathbb{Z}$ Suy ra $a=(k+1)(2k+3)$ Tương tự ta cũng có $2k+3=^+_- 1\vee k+1=^+_- 1$ Tuy nhiên  lúc này ta không nhận được giá trị $a, b, c$ nguyên tố nào.

Tóm lại $$\boxed{T=2015}$$

Monday, July 20, 2015

Bài toán: Tìm tất cả số nguyên dương $n$ sao cho $2^n-1$ chia hết cho $3$ và $\dfrac{2^n-1}{3}$ là ước của một số nguyên có dạng $4m^2+1$.

Lời giải:

Bổ đề 1: Với mỗi số nguyên dạng $4k+3$ thì luôn tồn tại một ước nguyên tố $p\equiv 3\pmod{4}$.
Chứng minh: 
Giả sử $n$ là số nguyên dạng $4k+3$. G/s luôn $n=p_1^{a_1}p_2^{a_2}...p_j^{a_j}$, với $p_i$ là các số nguyên tố, $a_i\in\mathbb{N^*}, i=\overline{1, k}$. Vì $n=4k+3$ nên $n$ là số lẻ. Nếu $p_i\equiv 1\pmod{4}$ thì $n\equiv 1\pmod{4}$, vô lý. Vậy phải tồn tại $p_l$ sao cho $p_l\equiv 3\pmod{4}$. Bổ đề 1 được chứng minh.

Bổ đề 2: Nếu $p$ là số nguyên tố dạng $4k+3$ và tồn tại $x, y\in\mathbb{Z}$ thỏa $p\;|\; x^2+y^2$ thì $\begin{cases}p\;|\; x\\p\;|\; y\end{cases}$
Chứng minh: 
 Ta xét trong TH $\begin{cases}p\;\not |\; x\\p\;\not |\; y\end{cases}$. Sử dụng định lý Fermat, ta có: $$\begin{cases} x^{p-1}\equiv 1\pmod{p}\\y^{p-1}\equiv 1\pmod{p}\end{cases}\Rightarrow \begin{cases}x^{4k+2}\equiv 1\pmod{p}\\y^{4k+2}\equiv 1\pmod{p}\end{cases}\Rightarrow x^{4k+2}+y^{4k+2}\equiv 2\pmod{p}$$ Điều này mâu thuẫ với giả thiết $p\;|\; x^2+y^2$. Vậy bổ đề 2 được chứng minh.

Quay lại bài toán:
Ta có: $2\equiv -1\pmod{3} $ nên $2^n\equiv 1\pmod{3}\Leftrightarrow n $ chẵn. Giả sử: $n=2^k.h,\; h$ lẻ. ta sẽ CM $h=1$. Thật vậy, g/s $h>1$. Ta có: $$2^h-1\;|\; 2^n-1\;|\; 3.4m^2+3$$ Do xét $h $ lẻ nên ta xét với $2^h-1\;|\; 4m^2+1$. Do $2^h-1\equiv 3\pmod{4}$ nên theo bổ đề 1, tồn tại một ước nguyên tố của $2^h -1$ dạng $2^l-1$ Sử dụng bổ đề 2, ta có: $$\begin{cases}2^l-1\; |\; 4m^2\\2^l-1\;|\;1\end{cases}\Leftrightarrow l=1, \text{vô lý}$$ Do vậy $n=2^k, k\in\mathbb{N}$. Ta sẽ CM đây cũng chính là KQ cần tìm. Ta phân tích: $$2^{2^k}-1=(2^{2^0}+1)(2^{2^1}+1)(2^{2^2}+1)...(2^{2^{k-1}}+1)$$ Chú ý rằng $3\;|\; 2^n-1$ nên ta xét với $\prod_{i=1}^{k-1}(2^{2^i}+1)$ Xét hệ thặng dư: $$\begin{cases} x\equiv 2^{2^0-1}\pmod{2^{2^1}+1}\\x\equiv 2^{2^1-1}\pmod{2^{2^{2}}+1}\\...\\x\equiv 2^{2^{i-1}-1}\pmod{2^{2^i}+1}\\...\\x\equiv 2^{2^{k-2}-1}\pmod{2^{2^{k-1}}+1}\end{cases}\Leftrightarrow \begin{cases}4x^2\equiv -1\pmod{2^{2^1}+1}\\4x^2\equiv -1 \pmod{2^{2^2}+1}\\...\\4x^2\equiv -1\pmod{2^{2^i}+1}\\...\\4x^2\equiv -1\pmod{2^{2^{k-1}}+1}\end{cases}$$. Chú ý rằng $\gcd(2^{2^i}+1; 2^{2^j}+1)=1, i\neq j$ nên theo định lý thặng dư Trung Hoa, hệ này có nghiệm duy nhất: $$4x^2\equiv -1\pmod{\prod_{i=1}^{k-1}(2^{2^i}+1)}\Leftrightarrow 4m^2+1\;\vdots \;\dfrac{2^n-1}{3}$$
Vậy: $n=2^k, k\in\mathbb{N}$

Saturday, July 18, 2015

HÀM LỒI, LÕM. HÀM BÁN LỒI, BÁN LÕM VÀ NGUYÊN LÝ BIÊN

HÀM LỒI, LÕM. HÀM BÁN LỒI, BÁN LÕM
VÀ NGUYÊN LÝ BIÊN

I. Định nghĩa:

    1. Hàm lồi, lõm:
       Hàm số $f$ được gọi là lồi trên đoạn $[\alpha; \beta ]\subset\mathbb{R}$ nếu với mọi $x, y\in [\alpha; \beta ]$ và với mọi $a,b\ge 0$ thỏa $a+b=1$ thì: $$f(ax+by)\ge af(x)+bf(y)$$ 
       Ngược lại, $f$ được gọi là lõm trên đoạn $[\alpha; \beta ]\subset\mathbb{R}$ nếu với mọi $x, y\in [\alpha; \beta ]$ và với mọi $a,b\ge 0$ thỏa $a+b=1$ thì: $$f(ax+by)\le af(x)+bf(y)$$ 

 Chú ý: Ta thừa nhận kết quả sau:

     Nếu hàm $f$ khả vi hai lần trên $[\alpha ;\beta ]$ thì $f$ lồi trên $[\alpha ;\beta ]$ khi $f''\le 0,\forall x\in [\alpha; \beta ]$. Ngược lại, $f$ lõm trên $[\alpha; \beta ] $ khi $f''\ge 0,\forall x\in [\alpha ;\beta ]$

    2. Hàm bán lồi, bán lõm:

       Hàm số $f$ được gọi là bán lồi, bán lõm trên $[\alpha ; \beta ]\subset \mathbb{R}$ nếu tồn tại hằng số $\gamma\in [\alpha ; \beta ]$ duy nhất sao cho $f$ lồi trên $[\alpha ; \gamma ]$ và lõm trên $[\gamma ; \beta ]$ hoặc ngược lại.

II. Tính chất:

                                                                         
    1. Nếu $f$ là hàm lồi trên $[\alpha ; \beta ]$ thì ta có: $\min f=\min\{ f(\alpha ); f(\beta )\}$
    2. Nếu $f$ lõm trên $[\alpha ; \beta ]$ thì ta có: $\max f=\max\{ f(\alpha ); f(\beta )\}$
    3. Nếu $f$ là một hàm lõm trên đoạn $[\alpha ; \beta ]$ thì khi đó với mọi $x,y, z\in\mathbb{R}$ thỏa mãn đồng thời $\beta \ge x\ge z\ge y\ge \alpha$ và $x+y-z\ge\alpha  $ thì $$f(x)+f(y)-f(z)\ge f(x+y-z)$$ 
 Chứng minh: 
 Đặt $t=x-z$, dễ thấy $0\le t\le x-y\Rightarrow t=k(x-y), k\in [0; 1]$. Ta có: $$\begin{aligned}f(x)+f(y)-f(z)-f(x+y-z)&=f(x)+f(y)-f(x-t)-f(y+t)\\&=f(x)+f(y)-f\left(x-k(x-y)\right)-f\left( y+k(x-y)\right)\\&=f(x)+f(y)-f\left((1-k)x+ky\right)-f\left( kx+(1-k)y\right) \\&\ge f(x)+f(y)-(1-k)f(x)-kf(y)-kf(x)-(1-k)f(y)=0\;\;\;\text{(Q.E.D)}\end{aligned}$$ 

   4. Nếu $f$ lồi trên  trên đoạn $[\alpha ; \beta ]$ thì khi đó với mọi $x,y, z\in\mathbb{R}$ thỏa mãn đồng thời $\beta \ge x\ge z\ge y\ge \alpha$ và $x+y-z\ge\alpha$ thì $$f(x)+f(y)-f(z)\le f(x+y-z)$$ 

   5. Nếu $f$ lõm trên $[\alpha ; \beta ]$ thì với dãy số $x_n \in [\alpha ; \beta ]$ thỏa $\sum_{i=1}^nx_i\le \beta+(n-1)\alpha$ thì khi đó: $$f(x_1)+f(x_2)+...+f(x_n)-(n-1)f(a)\le f(x_1+x_2+...+x_n-(n-1)\alpha )$$
 Chứng minh: 
 Ta chứng minh bằng qui nạp theo $n$ 
 Với $n=1$, hiển nhiên đúng.
 Giả sử tính chất 5 đúng tới $n$. Ta xét khi $n+1$.
 Không giảm tính tổng quát, giả sử $x_{n+1}=\max\{x_1; x_2;...; x_{n+1}\}$. Sử dụng giả thiết qui nạp ta chỉ cần chứng minh: $$f(x_1+x_2+...+x_n-(n-1)\alpha )+f(x_{n+1})\le f(\alpha )+f(x_1+x_2+...+x_{n+1}-n\alpha )$$ Do $x_{n+1}=\max\{x_1, x_2,..., x_{n+1}\}$ và $x_i\in [\alpha; \beta ]$ nên: $$\begin{aligned}f(\alpha )+f(x_1+x_2+...+x_{n+1}-n\alpha )&\ge f(x_1+x_2+...+x_{n+1}-n\alpha +\alpha -x_{n+1})+f(x_{n+1})\\&=f(x_1+x_2+...+x_n-(n-1)\alpha )+f(x_{n+1})\end{aligned}$$ Như vậy, khẳng định đúng với $n+1$. Ta có đpcm.

  6. Nếu $f$ lồi trên $[\alpha ; \beta ]$ thì với dãy số $x_n \in [\alpha ; \beta ]$ thỏa $\sum_{i=1}^nx_i\le \beta+(n-1)\alpha$ thì khi đó: $$f(x_1)+f(x_2)+...+f(x_n)-(n-1)f(a)\ge f(x_1+x_2+...+x_n-(n-1)\alpha )$$

 7. (BDT tiếp tuyến)
      $*$ Nếu $f$ lõm trên $[\alpha ; \beta ]$ thì với mọi $x, x_0\in [\alpha ; \beta ]$ ta có : $$f(x)\ge f'(x_0)(x-x_0)+f(x_0)$$
      $*$ Nếu $f$ lồi trên $[\alpha ; \beta ]$ thì với mọi $x, x_0\in [\alpha ; \beta ]$ ta có : $$f(x)\le f'(x_0)(x-x_0)+f(x_0)$$

  $*$ Đặc biệt, khi $f(x)=ax+b, a\neq 0$ thì đồ thị $f$ là một đoạn thẳng nên: $$\min\{f(\alpha ); f(\beta )\}\le f(x)\le \max\{f(\alpha ); f(\beta )\}$$

Định lý 1: Cho $(x_n)$ là dãy các số thực thỏa:
               i) $(x_n)$ là dãy không giảm.
               ii) $x_n\in [\alpha ; \beta ]$
               iii) $x_1+x_2+...+x_n=k=\text{const}$
               iv) $f$ là hàm trên $[\alpha ; \beta ]$ thỏa $f$ lồi trên $[\alpha ; \gamma ]$ và lõm trên$[\gamma ; \beta ]$.
Đặt $F=f(x_1)+f(x_2)+...+f(x_n)$. Khi đó: 
       + $\min F\Leftrightarrow x_1=x_2=...=x_{k-1}=a; x_{k+1}=...=x_n\in [\alpha ; \beta ]$
       + $\max F\Leftrightarrow x_1=x_2=...=x_{k-1}\in [\alpha ; \beta ]; x_{k+1}=...=x_n=b$

Định lý 2: Cho $(x_n)$ là dãy các số thực thỏa:
               i) $(x_n)$ là dãy không giảm.
               ii) $x_n\in [\alpha ; \beta ]$
               iii) $x_1+x_2+...+x_n=k=\text{const}$
               iv) $f$ là hàm trên $[\alpha ; \beta ]$ thỏa $f$ lõm trên $[\alpha ; \gamma ]$ và lồi trên$[\gamma ; \beta ]$.
Đặt $F=f(x_1)+f(x_2)+...+f(x_n)$. Khi đó: 
       + $\max F\Leftrightarrow x_1=x_2=...=x_{k-1}=a; x_{k+1}=...=x_n\in [\alpha ; \beta ]$
       + $\min F\Leftrightarrow x_1=x_2=...=x_{k-1}\in [\alpha ; \beta ]; x_{k+1}=...=x_n=b$

 Định lý 3: Cho $(x_n)$ là dãy các số thực thỏa:
               i) $(x_n)$ là dãy không giảm.
               ii) $x_n\in\mathbb{R}$
               iii) $x_1+x_2+...+x_n=k=\text{const}$
               iv) $f$ là hàm trên $[\alpha ; \beta ]$ thỏa $f$ lồi trên $(-\infty; \gamma ]$ và lõm trên$[\gamma ; +\infty )$.
Đặt $F=f(x_1)+f(x_2)+...+f(x_n)$. Khi đó: 
       + $\min F\Leftrightarrow x_1\le x_2=x_3=...x_n$
       + $\max F\Leftrightarrow x_1=x_2=...=x_{n-1}\le x_n$

Định lý 4: Cho $(x_n)$ là dãy các số thực thỏa:
               i) $(x_n)$ là dãy không giảm.
               ii) $x_n\in\mathbb{R}$
               iii) $x_1+x_2+...+x_n=k=\text{const}$
               iv) $f$ là hàm trên $[\alpha ; \beta ]$ thỏa $f$ lõm trên $(-\infty; \gamma ]$ và lồi trên$[\gamma ; +\infty )$.
Đặt $F=f(x_1)+f(x_2)+...+f(x_n)$. Khi đó: 
       + $\max F\Leftrightarrow x_1\le x_2=x_3=...x_n$
       + $\min F\Leftrightarrow x_1=x_2=...=x_{n-1}\le x_n$

II. Các ví dụ: 

 Ví dụ 1: (IMO Shorlist 1993, Viet Nam) Cho $a, b, c, d$ là các số thực không âm thỏa $a+b+c+d=1$. CMR: $$abc+abd+acd+bcd\le \dfrac{1}{27}+\dfrac{176}{27}abcd$$
Lời giải:
  Giả sử $a=\min\{a,b,c,d\}\Rightarrow a\le\dfrac{1}{4}$. BDT cần chứng minh viết lại dưới dạng: $$\left(bc+cd+bd-\dfrac{176}{27}bcd\right)a+bcd\le\dfrac{1}{27}\;\;\;\;\;\;\;\;\;\;(1)$$ Xem VT của $(1)$ như một hàm số theo $a$. Ta xét các trường hợp: 
      Trường hợp 1: $bc+cd+bd-\dfrac{176}{27}bcd=0.$ Khi đó:  $$VT_{(1)}=bcd\le\dfrac{(b+c+d)^3}{27}\le\dfrac{(a+b+c+d)^3}{27}=\dfrac{1}{27}$$
      Trường hợp 2: $bc+cd+bd-\dfrac{176}{27}bcd<0.$ Khi đó:  $$VT_{(1)}\le bcd\le \dfrac{1}{27}$$
      Trường hợp 3: $bc+cd+bd-\dfrac{176}{27}bcd>0.$ Khi đó: $$VT_{(1)}\le \dfrac{bc+cd+bd}{4}-\dfrac{17}{27}bcd$$ Sử dụng BDT Schur, ta có: $$\dfrac{bc+cd+bd}{4}-\dfrac{17}{27}bcd\le \dfrac{13}{324}(bc+cd+bd)+\dfrac{17}{576}\le \dfrac{13(b+c+d)^2}{324.3}+\dfrac{17}{576}=\dfrac{1}{27}$$
 Phép chứng minh hoàn tất.
    Lưu ý: Tùy theo bài toán ra cần lựa đại lượng thay đổi phù hợp sao cho thuận lợi trong việc công phá bài toán. Đối với bài toán này, ta có thể cố định $c, d$ thì do $a+b+c+d=1$ nên $a+b$ cũng cố định, lúc này khảo sát $VT_{(1)}$ theo $ab$ ta cũng thu dược đpcm.

Ví dụ 2: (Vasile Cirtoaje) Cho $a, b, c, d$ là các số thực không âm $a, b, c, d$ thỏa $a^2+b^2+c^2+d^2=1$. CMR: $$(1-a)(1-b)(1-c)(1-d)\ge abcd$$
Lời giải:
 Giả sử $a=\min\{a,b,c,d\}\Rightarrow a\le\dfrac{1}{2}$. Ta viết BDT cần chứng minh dưới dạng: $$[(1-b)(1-c)(1-d)+bcd]a-(1-b)(1-c)(1-d)\le 0\;\;\;\;\;\;\;\;\;\;\;\;\;(1)$$ Xem VT của $(1)$ như một hàm theo $a$. Ta có hai trường hợp:
     Trường hợp 1: $(1-b)(1-c)(1-d)+bcd=0$, hiển nhiên đúng.
     Trường hợp 2: $(1-b)(1-c)(1-d)+bcd>0$. Khi đó: $$[(1-b)(1-c)(1-d)+bcd]a-(1-b)(1-c)(1-d)\le \dfrac{bcd-(1-b)(1-c)(1-d)}{2}$$ Đặt $\begin{cases}p=b+c+d\\q=bc+cd+db\\r=bcd\end{cases}$. Khi đó, ta cần CM:$$\dfrac{2r+p-q-1}{2}\le 0$$ Chú ý là lúc này $p^2-2q=\dfrac{3}{4}$ nên ta cần CM: $$4r-\dfrac{1}{4}\le (p-1)^2$$ Theo AM-GM: $p\ge 3\sqrt[3]{r}$ Từ đây chỉ việc khảo sát hàm $g ( r ) =4r-9\sqrt[3]{r^2}+6\sqrt{r}-\dfrac{5}{4}, r\le \dfrac{1}{8}$ là có đpcm. 


 Ví dụ 3: (Crux Mathemmaticorum) Cho $a,b,c$ là các số không âm. CMR: $$\sqrt{1+\dfrac{48a}{b+c}}+\sqrt{1+\dfrac{48b}{c+a}}+\sqrt{1+\dfrac{48c}{a+b}}\ge 15$$ Lời giải:

Đặt $x=\dfrac{2a}{b+c}; y=\dfrac{2b}{c+a}; z=\dfrac{2c}{a+b}$. Khi đó, ta có: $$xy+yz+zx+xyz=4$$ Và ta cần CM: $$\sqrt{1+24x}+\sqrt{1+24y}+\sqrt{1+24z}\ge 15\;\;\;\;\;\;\;\;\;\;\;\;\;\;\;(1)$$ Không giảm tính tổng quát, giả sử: $x=\max\{ x,y, z\}\Rightarrow x\ge 1$. Xem VT của $(1)$ như một hàm $f$ theo $x$. Dễ thấy $f$ đồng biến nên $f$ đạt giá trị nhỏ nhất khi $x=1$. Lúc này, ta có $y+z+2yz=4\Leftrightarrow y=\dfrac{4-z}{1+2z}$ và ta cần tìm GTNN của $$\sqrt{1+24y}+\sqrt{1+24z}=\sqrt{1+24.\dfrac{4-z}{1+2z}}+\sqrt{1+24z}$$ Thực hiện khảo sát hàm $g(z)=\sqrt{1+24.\dfrac{4-z}{1+2z}}+\sqrt{1+24z}$ với $z\in [0; 1]$ ta dễ dàng thấy $g(z)\ge 10$ Từ đó suy ra: $$\sqrt{1+24x}+\sqrt{1+24y}+\sqrt{1+24z}\ge 15\;\text{  (Q.E.D)}$$

Ví dụ 4: Cho các số $a,b,c$ không âm. CMR: $$\left( \dfrac{4a}{b+c}+1\right)\left( \dfrac{4b}{c+a}+1\right)\left(\dfrac{4c}{a+b}+1\right)\ge 25$$ Lời giải:

Đặt $x=\dfrac{2a}{b+c}, y=\dfrac{2b}{c+a}, z=\dfrac{2c}{a+b}$. Khi đó: $$xy+yz+zx+xyz=4$$
Và ta cần CM: $$(1+2x)(1+2y)(1+2z)\ge 25$$Nếu ta xem VT của BDT này như một hàm theo $z$ thì rõ ràng đây là một hàm bậc nhất và đồng biến. Theo đó. GTNN của hàm này đạt được khi $z=0$. Lúc này $xy=4$ và ta cần chỉ ra: $$(1+2x)(1+2y)\ge 25\Leftrightarrow x+y\ge 8$$ Điều này là hiển nhiên theo AM-GM. Bài toán được chứng minh.

Ví dụ 5: (USA MO 2000) Cho $a,b, c$ là các số thực không âm. CMR: $$\dfrac{a+b+c}{3}-\sqrt[3]{abc}\le\max\{(\sqrt{a}-\sqrt{b})^2; (\sqrt{b}-\sqrt{c})^2; (\sqrt{c}-\sqrt{a})^2\}$$ Lời giải:

Không mất tính tổng quát, giả sử $a\le b\le c$. Khi đó, ta cần CM: $$a+b+c-3\sqrt[3]{abc}-3(\sqrt{c}-\sqrt{a})^2\ge 0\;\;\;\;\;\;\;\;\;\;\;\;\;\;\;\;\;(1)$$ Xem VT của $(1)$ là một hàm theo  $b$. Ta tính được $$f''(b)=\dfrac{\sqrt[3]{ca}}{3\sqrt[3]{b^5}}>0$$
 Do đó $f$ là hàm lõm. Như vậy GTLN của $f$ đạt được khi $b=a$ hoặc $b=c$. Ta xét khi $a=b$. Khi đó ta cần CM: $$a+2c+3\sqrt[3]{ca^2}\ge 6\sqrt{ca}$$ Điều này là đúng theo AM-GM.

Nếu mở rộng BDT trên ta tìm được chặn dưới của đại lượng TBC-TBN là $\dfrac{1}{2}\min_{1\le i<j\le n}\{(\sqrt{a_i}-\sqrt{a_j})^2\}$. Đây chính là bài toán BDT trong kì thi MOSP 2000: 

 Cho $a_1, a_2,...,a_n$ là các số không âm. CMR: $$\dfrac{1}{n}\sum_{i=1}^n a_i-\sqrt[n]{\prod_{i=1}^n a_i}\ge \dfrac{1}{2}\min_{1\le i<j\le n}\{(\sqrt{a_i}-\sqrt{a_j})^2\}$$ 

Ví dụ 6: Cho $a, b, c\ge 0$ có tổng bằng $1$. CMR: $$\sqrt{\dfrac{1-a}{1+a}}+\sqrt{\dfrac{1-b}{1+b}}+\sqrt{\dfrac{1-c}{1+c}}\le 1+\dfrac{2}{\sqrt{3}}$$ Lời giải: 
Giả sử $a\le b\le c$. Xét hàm số: $$ f(x)=\sqrt{\dfrac{1-x}{1+x}}, x\in [0; 1]$$ Ta tính được: $$f''(x)=\dfrac{1-2x}{(x+1)^4\sqrt{\left(\dfrac{1-x}{1+x}\right)^3}}$$ Ta thấy $f''$ đổi dấu khi qua $x=1/2$ nên $f$ lõm trên $\left[0; \dfrac{1}{2}\right]$ và lồi trên $\left[\dfrac{1}{2}; 1\right]$. Do đó theo định lý 2, VT đạt cực đại khi: $$\begin{cases}a=0, b=c\\a+b+c=1\end{cases}\vee \begin{cases}a=b=0\\c=1\end{cases}$$ Giải các hệ này ta dễ dàng suy ra đpcm.

Tổng quát hóa bài toán ví dụ 6, ta thu được bài toán của tạp chí Crux Mathemmaticorum: 
Cho $x_1, x_2,..., x_n\;\; (n\ge 2)$ là các số không âm có tổng bằng $1$. CMR: $$\sqrt{\dfrac{1-x_1}{1+x_1}}+\sqrt{\dfrac{1-x_2}{1+x_2}}+...+\sqrt{\dfrac{1-x_n}{1+x_n}}\le n-2+\dfrac{2}{\sqrt{3}}$$

Ví dụ 7: (VMEO 2005) Cho $a,b,c$ là các số thực dương cho trước và $x, y, z$ là các số thực dương thỏa mãn $ax+by+cz=xyz$. Tìm GTNN của: $$P=x+y+z$$ Lời giải:
Bổ đề: Với các số thực dương $a, b, c$ cho trước. Khi đó tồn tại duy nhất $d>0$ sao cho: $$\dfrac{1}{a+d}+\dfrac{1}{b+d}+\dfrac{1}{c+d}=\dfrac{2}{d}$$ Chứng minh bổ đề: Viết biểu thức trên lại thành: $$\dfrac{a}{a+d}+\dfrac{b}{b+d}+\dfrac{c}{c+d}=1$$ Xem đây là một hàm $f$ theo $d$. Dễ thấy $f$ liên tục và $f$ nghịch biến. Mặt khác $\lim_{d\to 0}f(d) =3$ và $\lim_{d\to +\infty } f(d)=0$ nên tồn tại duy nhất số $d>0$ thỏa mãn đẳng thức trên. Bổ đề được chứng minh.

Quay lại bài toán. Đặt $x=\sqrt{\dfrac{bc}{a}}m, y=\sqrt{\dfrac{ca}{b}}n, z=\sqrt{\dfrac{ab}{c}}p$. Khi đó ta có: $$m+n+p=mnp$$ Điều này gợi cho ta chọn $A, B, C$ là ba góc của một tam giác nhọn sao cho: $m=\tan A, n=\tan B, p=\tan C$. Bây giờ, ta cần tìm GTNN của: $$P=\sqrt{\dfrac{bc}{a}}\tan A+\sqrt{\dfrac{ca}{b}}\tan B+\sqrt{\dfrac{ca}{b}}\tan C$$ Xét hàm $f(x)=\tan x, x\in \left(0;\dfrac{\pi}{2}\right)$. Ta có : $f"(x)=2\tan x. (\tan ^2 x+1)>0,\forall x\in\left(0, \dfrac{\pi}{2}\right)$ nên $f$ lõm trên $\left(0; \dfrac{\pi}{2}\right)$. Ta chọn $d$ là số thực dương sao cho: $$\dfrac{1}{a+d}+\dfrac{1}{b+d}+\dfrac{1}{c+d}=\dfrac{2}{d}$$ Sự tồn tại của $d$ hoàn toàn được khẳng định theo bổ đề. Áp dụng BDT tiếp tuyến, ta có: $$\begin{aligned} & f(A)\ge f'\left(\arctan\sqrt{\dfrac{a(b+d)(c+d)}{bc(a+d)}}\right)\left[A-\arctan\sqrt{\dfrac{a(b+d)(c+d)}{bc(a+d)}}\right]+f\left(\arctan\sqrt{\dfrac{a(b+d)(c+d)}{bc(a+d)}}\right)\\\Rightarrow &\tan A\ge \dfrac{a(b+d)(c+d)+bc(a+d)}{bc(a+d)}\left[A-\arctan\sqrt{\dfrac{a(b+d)(c+d)}{bc(a+d)}}\right]+\sqrt{\dfrac{a(b+d)(c+d)}{bc(a+d)}}\\&\Rightarrow\sqrt{\dfrac{bc}{a}}\tan A\ge \dfrac{2abc+(ab+bc+ca)d+ad^2}{\sqrt{abc}(a+d)}\left[A-\arctan\sqrt{\dfrac{a(b+d)(c+d)}{bc(a+d)}}\right]+\sqrt{\dfrac{(b+d)(c+d)}{a+d}}\end{aligned}$$ Chú ý rằng vì $$\dfrac{1}{a+d}+\dfrac{1}{b+d}+\dfrac{1}{c+d}=\dfrac{2}{d}$$ nên ta có ngay  $$d^3=2abc+(ab+bc+ca)d$$ Suy ra: $$\sqrt{\dfrac{bc}{a}}\tan A\ge\dfrac{d^2}{\sqrt{abc}}\left[ A-\arctan\sqrt{\dfrac{a(b+d)(c+d)}{bc(a+d)}}\right] +\sqrt{\dfrac{(b+d)(c+d)}{a+d}}$$ Tương tự, ta cũng có: $$\sqrt{\dfrac{ca}{b}}\tan B\ge \dfrac{d^2}{\sqrt{abc}}\left[ B-\arctan\sqrt{\dfrac{b(a+d)(c+d)}{ac(b+d)}}\right] +\sqrt{\dfrac{(a+d)(c+d)}{b+d}}\\\sqrt{\dfrac{ab}{c}}\tan C\ge \dfrac{d^2}{\sqrt{abc}}\left[ C-\arctan\sqrt{\dfrac{c(a+d)(b+d)}{ab(c+d)}}\right] +\sqrt{\dfrac{(a+d)(b+d)}{c+d}}$$ Chú ý rằng: $$\sqrt{\dfrac{a(b+d)(c+d)}{bc(a+d)}}+\sqrt{\dfrac{b(c+d)(a+d)}{ca(b+d)}}+\sqrt{\dfrac{c(a+d)(b+d)}{ab(c+d)}}=\sqrt{\dfrac{(a+d)(b+d)(c+d)}{abc}}$$ nên: $$A+B+C-\arctan\sqrt{\dfrac{a(b+d)(c+d)}{bc(a+d)}}-\arctan\sqrt{\dfrac{b(c+d)(a+d)}{ca(b+d)}}+\arctan\sqrt{\dfrac{c(a+d)(b+d)}{ab(c+d)}}=0$$ Do đó: $$P\ge \sqrt{\dfrac{(a+d)(b+d)}{c+d}}+\sqrt{\dfrac{(b+d)(c+d)}{a+d}}+\sqrt{\dfrac{(c+d)(a+d)}{b+d}}$$ Vậy: $$\min P=\sqrt{\dfrac{(b+d)(c+d)}{a+d}}+\sqrt{\dfrac{(c+d)(a+d)}{b+d}}+\sqrt{\dfrac{(a+d)(b+d)}{c+d}}$$

Kết thúc bài viết, tôi xin đề nghị một số bài tập tự luyện: 
 1. Cho $a, b, c, d\in [0; 1]$. CMR: $$(1-a)(1-b)(1-c)(1-d)+a+b+c+d\ge 1$$
 2. (THTT 2002) Cho $a, b, c$ là các số thực có tổng bình phương bằng $2$. Tìm GTLN của $$P=3(a+b+c)-22abc$$
 3. Cho $a, b, c, d\ge 0$ thỏa $a+b+c+d=4$. CMR: $$16+2abcd\ge 3(ab+ac+ad+bc+bd+cd)$$
 4. (Viet Nam TST 2001, Trần Nam Dũng) Cho $a, b, c$ là các số dương thỏa $21ab+2bc+8ca\le 12$. Tìm GTNN của: $$P=\dfrac{1}{a}+\dfrac{2}{b}+\dfrac{3}{c}$$
 5. (IMO 1984) Cho $x,y,z\ge 0$ thỏa $x+y+z=1$. CMR: $$0\ge xy+yz+zx-2xyz\le \dfrac{7}{27}$$
 6. Cho $x,y,z>0$ thỏa $x+y+z=1$. Tìm GTLN của $$F=\dfrac{x}{x^2+1}+\dfrac{y}{y^2+1}+\dfrac{z}{z^2+1}$$
 7. (VMEO 2004) Cho tam giác nhọn $ABC$. Tìm GTNN của: $$P=\tan A+2\tan B+5\tan C$$
 8. Cho $x, y, z, t\ge 0$ thỏa $x+y+z+t=4$. CMR: $$(1+x)(1+3y)(1+3z)(1+3t)\le 125 +131xyzt$$

Tài liệu tham khảo:
 $[1]$. Bất đẳng thức- Võ Quốc Bá Cẩn.
 $[2]$. Kỹ thuật đánh giá trên biên - Phạm Quốc Thắng.
 $[3]$. Nguyên lý biên - Cao Đình Huy.
 $[4]$. www.diendantoanhoc.net

Monday, July 13, 2015

ĐỀ THI IMO 2015



IMO 2015
Ngày 10-07-2015

Bài 1. Cho $S$ là một tập hữu hạn các điểm trên mặt phẳng. Tập $S$ được gọi là "cân bằng" nếu với với hai điểm $A,B$ phân biệt bất kì thuộc $S$ thì luôn tồn tại điểm $C$ thuộc $S$ thoả mãn $AC=BC$. Tập $S$ được gọi là "không tâm" nếu với bất kì ba điểm phân biệt $A,B,C$ thuộc $S$ thì không có điểm $P$ nào thoả mãn $PA=PB=PC$.
     a) Chứng minh rằng với mọi số nguyên $n \ge 3$, tồn tại một tập "cân bằng" với $n$ điểm.
     b) Tìm tất cả số nguyên $n \ge 3$, sao cho tồn tại một tập "cân bằng" và "không tâm" cho $n$ điểm.

Bài 2. Tìm mọi số nguyên dương $(a,b,c)$ thoả mãn $ab-c,bc-a,ca-b$ đều là luỹ thừa của $2$.

Bài 3. Cho tam giác nhọn $ABC$, $AB>AC$ có đường tròn ngoại tiếp $\Gamma$, trực tâm $H$ và chân đường cao $F$ hạ từ $A$. $M$ là trung điểm $BC$. $Q$ là điểm trên $\Gamma$ thoả mãn $\angle HQA= 90^{\circ}$, và $K$ là điểm trên $\Gamma$ sao cho $\angle HKQ=90^{\circ}$. $A,B,C,K,Q$ là các điểm phân biệt, và chúng nằm trên $\Gamma$ theo đúng thứ tự đó. Chứng minh rằng đường tròn ngoai tiếp hai tam giác $KQH$ và $FKM$ tiếp xúc với nhau.


IMO 2015
Ngày 11-07-2015


Bài 4. Tam giác $ABC$ nội tiếp đường tròn $\Omega$ có tâm $O$. Một đường tròn $\Gamma$ với tâm $A$ cắt cạnh $BC$ tại $D$ và $E$ sao cho $B,D,E,C$ phân biệt và nằm trên đường thẳng $BC$ theo đúng thứ tự này. $F,G$ là giao điểm của $\Omega$ và $\Gamma$ sao cho $A,F,B,C,G$ nằm trên $\Omega$ theo đúng thứ tự này. $K$ là giao điểm thứ hai của đường tròn ngoại tiếp $\triangle BDF$ và cạnh $AB$. $L$ là giao điểm thứ hai của đường tròn ngoại tiếp $\triangle CGE$ và cạnh $CA$.

Giả sử đường thẳng $FK$ và $GL$ phân biệt và cắt nhau tại $X$. Chứng minh rằng $X$ nằm trên đường thẳng $AO$.

Bài 5. Kí hiệu $\mathbb{R}$ là tập các số thực. Xác định tất cả các hàm số $f: \mathbb{R} \to \mathbb{R}$ thoả mãn $$f(x+f(x+y))+f(xy)=x+f(x+y)+yf(x)$$ với mọi số thực $x,y$.

Bài 6. Cho dãy $a_1,a_2, \cdots $ các số nguyên thoả mãn điều kiện sau:
     (i) $1 \le a_j \le 2015$ với mọi $j \ge 1$;
     (ii) $k+a_k \ne l+a_l$ với mọi $1 \le k<l$.

Chứng minh rằng tồn tại hai số nguyên dương $b$ và $N$ thoả mãn $$\left | \sum_{j=m+1}^n(a_j-b) \right | \le 1007^2$$ với mọi số nguyên $m,n$ thoả mãn $n>m \ge N$.

Nguồn: diendantoanhoc.net

Sunday, July 12, 2015

Bài toán [Nguyễn Văn Huyện]: Cho $a,b,c$ là các thực thỏa $a+b, b+c, c+a$ là các số dương. Chứng minh rằng: $$9(a+b)(b+c)(c+a)\ge 8(a+b+c)(ab+bc+ca)$$
Lời giải:
Do $\min\{a+b; b+c; c+a\}>0$ nên trong ba số $a, b, c$ phải có ít nhất hai số không âm. Giả sử đó là $b, c$. Bất đẳng thức cần chứng minh tương đương với: $$(a+b+c)(ab+bc+ca)\ge 3abc\\\Leftrightarrow (b+c)a^2+[(b+c)^2-8bc]a+bc(b+c)\ge 0$$ Vì $b+c>0$, xét biệt thức $$\Delta =[(b+c)^2-8bc]^2-4bc(b+c)^2=(b+c)^4-20bc(b+c)^2+64b^2c^2$$
Ta sẽ CM $\Delta \le 0$, tức là: $$(b+c)^4-20bc(b+c)^2+64b^2c^2\le 0$$ Chuẩn hóa cho $b+c=1\Rightarrow bc\le \dfrac{1}{4}$. Khi đó ta cần chứng minh: $$64b^2c^2-20bc+1\le 0$$ hiển nhiên đúng với $0<bc\le \dfrac{1}{4}$. Bài toán được giải quyết.